Uniform distribution on unit disk











up vote
3
down vote

favorite
1













Let $(X, Y)$ be a random point chosen according to the uniform distribution in the disk of radius 1 centered at the origin. Compute the densities of $X$ and of $Y$.




I know that the joint density of $X$ and $Y$ is $frac{1}{pi}$ since when we integrate $frac{1}{pi}$ over the unit circle, we get $1$.



So if I wanted to find the density of $X$, I was thinking of finding the cumulative distribution of $X$ and the differentiate it to get its density. In order to get its cumulative distribution function, I was going to use the fact that $P(X<x)=P(X<x, -infty < Y < infty)$, but this integral doesn't seem nice to work with. Am I on the right track or is there a better way?










share|cite|improve this question




























    up vote
    3
    down vote

    favorite
    1













    Let $(X, Y)$ be a random point chosen according to the uniform distribution in the disk of radius 1 centered at the origin. Compute the densities of $X$ and of $Y$.




    I know that the joint density of $X$ and $Y$ is $frac{1}{pi}$ since when we integrate $frac{1}{pi}$ over the unit circle, we get $1$.



    So if I wanted to find the density of $X$, I was thinking of finding the cumulative distribution of $X$ and the differentiate it to get its density. In order to get its cumulative distribution function, I was going to use the fact that $P(X<x)=P(X<x, -infty < Y < infty)$, but this integral doesn't seem nice to work with. Am I on the right track or is there a better way?










    share|cite|improve this question


























      up vote
      3
      down vote

      favorite
      1









      up vote
      3
      down vote

      favorite
      1






      1






      Let $(X, Y)$ be a random point chosen according to the uniform distribution in the disk of radius 1 centered at the origin. Compute the densities of $X$ and of $Y$.




      I know that the joint density of $X$ and $Y$ is $frac{1}{pi}$ since when we integrate $frac{1}{pi}$ over the unit circle, we get $1$.



      So if I wanted to find the density of $X$, I was thinking of finding the cumulative distribution of $X$ and the differentiate it to get its density. In order to get its cumulative distribution function, I was going to use the fact that $P(X<x)=P(X<x, -infty < Y < infty)$, but this integral doesn't seem nice to work with. Am I on the right track or is there a better way?










      share|cite|improve this question
















      Let $(X, Y)$ be a random point chosen according to the uniform distribution in the disk of radius 1 centered at the origin. Compute the densities of $X$ and of $Y$.




      I know that the joint density of $X$ and $Y$ is $frac{1}{pi}$ since when we integrate $frac{1}{pi}$ over the unit circle, we get $1$.



      So if I wanted to find the density of $X$, I was thinking of finding the cumulative distribution of $X$ and the differentiate it to get its density. In order to get its cumulative distribution function, I was going to use the fact that $P(X<x)=P(X<x, -infty < Y < infty)$, but this integral doesn't seem nice to work with. Am I on the right track or is there a better way?







      probability probability-distributions






      share|cite|improve this question















      share|cite|improve this question













      share|cite|improve this question




      share|cite|improve this question








      edited Apr 11 '15 at 18:30









      Did

      244k23212451




      244k23212451










      asked Jan 6 '15 at 4:56









      DHH

      163110




      163110






















          2 Answers
          2






          active

          oldest

          votes

















          up vote
          3
          down vote













          By definition, the marginal density of $X$ is simply $$f_X(x) = int_{y=-1}^1 f_{X,Y}(x,y) , dy = int_{y=-sqrt{1-x^2}}^{sqrt{1-x^2}} frac{1}{pi} , dy.$$ The second equality arises from the fact that $$f_{X,Y}(x,y) = frac{1}{pi} mathbb{1}(x^2+y^2 le 1),$$ from which we see that for a given $X = x$, the support of $Y$ is then $-sqrt{1-x^2} le Y le sqrt{1-x^2}$.






          share|cite|improve this answer




























            up vote
            1
            down vote













            $P(X<x)=int_{-1}^xint_{-sqrt{1-x^2}}^{sqrt{1-x^2}}frac{1}{pi}text{d}ytext{ d}x=int_{-1}^xfrac{1}{pi}2sqrt{1-x^2}text{d}x$, so differentiating you just get back the inside $f_X(x)=frac{1}{pi}2sqrt{1-x^2}$.






            share|cite|improve this answer























            • @DHH Sorry, I first thought you had a uniform distribution on the circle (only the outline) and not the disk. I've edited for the disk.
              – user1537366
              Jan 6 '15 at 5:38













            Your Answer





            StackExchange.ifUsing("editor", function () {
            return StackExchange.using("mathjaxEditing", function () {
            StackExchange.MarkdownEditor.creationCallbacks.add(function (editor, postfix) {
            StackExchange.mathjaxEditing.prepareWmdForMathJax(editor, postfix, [["$", "$"], ["\\(","\\)"]]);
            });
            });
            }, "mathjax-editing");

            StackExchange.ready(function() {
            var channelOptions = {
            tags: "".split(" "),
            id: "69"
            };
            initTagRenderer("".split(" "), "".split(" "), channelOptions);

            StackExchange.using("externalEditor", function() {
            // Have to fire editor after snippets, if snippets enabled
            if (StackExchange.settings.snippets.snippetsEnabled) {
            StackExchange.using("snippets", function() {
            createEditor();
            });
            }
            else {
            createEditor();
            }
            });

            function createEditor() {
            StackExchange.prepareEditor({
            heartbeatType: 'answer',
            convertImagesToLinks: true,
            noModals: true,
            showLowRepImageUploadWarning: true,
            reputationToPostImages: 10,
            bindNavPrevention: true,
            postfix: "",
            imageUploader: {
            brandingHtml: "Powered by u003ca class="icon-imgur-white" href="https://imgur.com/"u003eu003c/au003e",
            contentPolicyHtml: "User contributions licensed under u003ca href="https://creativecommons.org/licenses/by-sa/3.0/"u003ecc by-sa 3.0 with attribution requiredu003c/au003e u003ca href="https://stackoverflow.com/legal/content-policy"u003e(content policy)u003c/au003e",
            allowUrls: true
            },
            noCode: true, onDemand: true,
            discardSelector: ".discard-answer"
            ,immediatelyShowMarkdownHelp:true
            });


            }
            });














             

            draft saved


            draft discarded


















            StackExchange.ready(
            function () {
            StackExchange.openid.initPostLogin('.new-post-login', 'https%3a%2f%2fmath.stackexchange.com%2fquestions%2f1092669%2funiform-distribution-on-unit-disk%23new-answer', 'question_page');
            }
            );

            Post as a guest















            Required, but never shown

























            2 Answers
            2






            active

            oldest

            votes








            2 Answers
            2






            active

            oldest

            votes









            active

            oldest

            votes






            active

            oldest

            votes








            up vote
            3
            down vote













            By definition, the marginal density of $X$ is simply $$f_X(x) = int_{y=-1}^1 f_{X,Y}(x,y) , dy = int_{y=-sqrt{1-x^2}}^{sqrt{1-x^2}} frac{1}{pi} , dy.$$ The second equality arises from the fact that $$f_{X,Y}(x,y) = frac{1}{pi} mathbb{1}(x^2+y^2 le 1),$$ from which we see that for a given $X = x$, the support of $Y$ is then $-sqrt{1-x^2} le Y le sqrt{1-x^2}$.






            share|cite|improve this answer

























              up vote
              3
              down vote













              By definition, the marginal density of $X$ is simply $$f_X(x) = int_{y=-1}^1 f_{X,Y}(x,y) , dy = int_{y=-sqrt{1-x^2}}^{sqrt{1-x^2}} frac{1}{pi} , dy.$$ The second equality arises from the fact that $$f_{X,Y}(x,y) = frac{1}{pi} mathbb{1}(x^2+y^2 le 1),$$ from which we see that for a given $X = x$, the support of $Y$ is then $-sqrt{1-x^2} le Y le sqrt{1-x^2}$.






              share|cite|improve this answer























                up vote
                3
                down vote










                up vote
                3
                down vote









                By definition, the marginal density of $X$ is simply $$f_X(x) = int_{y=-1}^1 f_{X,Y}(x,y) , dy = int_{y=-sqrt{1-x^2}}^{sqrt{1-x^2}} frac{1}{pi} , dy.$$ The second equality arises from the fact that $$f_{X,Y}(x,y) = frac{1}{pi} mathbb{1}(x^2+y^2 le 1),$$ from which we see that for a given $X = x$, the support of $Y$ is then $-sqrt{1-x^2} le Y le sqrt{1-x^2}$.






                share|cite|improve this answer












                By definition, the marginal density of $X$ is simply $$f_X(x) = int_{y=-1}^1 f_{X,Y}(x,y) , dy = int_{y=-sqrt{1-x^2}}^{sqrt{1-x^2}} frac{1}{pi} , dy.$$ The second equality arises from the fact that $$f_{X,Y}(x,y) = frac{1}{pi} mathbb{1}(x^2+y^2 le 1),$$ from which we see that for a given $X = x$, the support of $Y$ is then $-sqrt{1-x^2} le Y le sqrt{1-x^2}$.







                share|cite|improve this answer












                share|cite|improve this answer



                share|cite|improve this answer










                answered Jan 6 '15 at 5:05









                heropup

                61.9k65997




                61.9k65997






















                    up vote
                    1
                    down vote













                    $P(X<x)=int_{-1}^xint_{-sqrt{1-x^2}}^{sqrt{1-x^2}}frac{1}{pi}text{d}ytext{ d}x=int_{-1}^xfrac{1}{pi}2sqrt{1-x^2}text{d}x$, so differentiating you just get back the inside $f_X(x)=frac{1}{pi}2sqrt{1-x^2}$.






                    share|cite|improve this answer























                    • @DHH Sorry, I first thought you had a uniform distribution on the circle (only the outline) and not the disk. I've edited for the disk.
                      – user1537366
                      Jan 6 '15 at 5:38

















                    up vote
                    1
                    down vote













                    $P(X<x)=int_{-1}^xint_{-sqrt{1-x^2}}^{sqrt{1-x^2}}frac{1}{pi}text{d}ytext{ d}x=int_{-1}^xfrac{1}{pi}2sqrt{1-x^2}text{d}x$, so differentiating you just get back the inside $f_X(x)=frac{1}{pi}2sqrt{1-x^2}$.






                    share|cite|improve this answer























                    • @DHH Sorry, I first thought you had a uniform distribution on the circle (only the outline) and not the disk. I've edited for the disk.
                      – user1537366
                      Jan 6 '15 at 5:38















                    up vote
                    1
                    down vote










                    up vote
                    1
                    down vote









                    $P(X<x)=int_{-1}^xint_{-sqrt{1-x^2}}^{sqrt{1-x^2}}frac{1}{pi}text{d}ytext{ d}x=int_{-1}^xfrac{1}{pi}2sqrt{1-x^2}text{d}x$, so differentiating you just get back the inside $f_X(x)=frac{1}{pi}2sqrt{1-x^2}$.






                    share|cite|improve this answer














                    $P(X<x)=int_{-1}^xint_{-sqrt{1-x^2}}^{sqrt{1-x^2}}frac{1}{pi}text{d}ytext{ d}x=int_{-1}^xfrac{1}{pi}2sqrt{1-x^2}text{d}x$, so differentiating you just get back the inside $f_X(x)=frac{1}{pi}2sqrt{1-x^2}$.







                    share|cite|improve this answer














                    share|cite|improve this answer



                    share|cite|improve this answer








                    edited Jan 6 '15 at 5:36

























                    answered Jan 6 '15 at 5:22









                    user1537366

                    1,519819




                    1,519819












                    • @DHH Sorry, I first thought you had a uniform distribution on the circle (only the outline) and not the disk. I've edited for the disk.
                      – user1537366
                      Jan 6 '15 at 5:38




















                    • @DHH Sorry, I first thought you had a uniform distribution on the circle (only the outline) and not the disk. I've edited for the disk.
                      – user1537366
                      Jan 6 '15 at 5:38


















                    @DHH Sorry, I first thought you had a uniform distribution on the circle (only the outline) and not the disk. I've edited for the disk.
                    – user1537366
                    Jan 6 '15 at 5:38






                    @DHH Sorry, I first thought you had a uniform distribution on the circle (only the outline) and not the disk. I've edited for the disk.
                    – user1537366
                    Jan 6 '15 at 5:38




















                     

                    draft saved


                    draft discarded



















































                     


                    draft saved


                    draft discarded














                    StackExchange.ready(
                    function () {
                    StackExchange.openid.initPostLogin('.new-post-login', 'https%3a%2f%2fmath.stackexchange.com%2fquestions%2f1092669%2funiform-distribution-on-unit-disk%23new-answer', 'question_page');
                    }
                    );

                    Post as a guest















                    Required, but never shown





















































                    Required, but never shown














                    Required, but never shown












                    Required, but never shown







                    Required, but never shown

































                    Required, but never shown














                    Required, but never shown












                    Required, but never shown







                    Required, but never shown







                    Popular posts from this blog

                    QoS: MAC-Priority for clients behind a repeater

                    Ивакино (Тотемский район)

                    Can't locate Autom4te/ChannelDefs.pm in @INC (when it definitely is there)